Đến nội dung

ZzNightWalkerZz nội dung

Có 155 mục bởi ZzNightWalkerZz (Tìm giới hạn từ 29-03-2020)



Sắp theo                Sắp xếp  

#621356 Tìm Min $\sum a^2 + \sum \frac{1}{(a-b)^2...

Đã gửi bởi ZzNightWalkerZz on 20-03-2016 - 10:13 trong Bất đẳng thức và cực trị

Giải bài toán gốc trước

Đặt

$f(a,b,c)=\sum a^2 . \sum \frac{1}{(a-b)^2} $

Ta thấy $f(a,b,c) \geq f(a-c,b-c;0)$

Do đó, ta chỉ cần tìm min trong TH $c=0$

Thay vào, tìm được min P

Còn bài toán đầu thì chỉ cần AM-GM là ra lại bài toán gốc

quan trọng là dấu bằng nên không AM-GM để ra bài toán gốc được :))

Dù sao lời giải cho bài toán gốc của bạn là đúng rồi




#621342 Tìm Min $\sum a^2 + \sum \frac{1}{(a-b)^2...

Đã gửi bởi ZzNightWalkerZz on 20-03-2016 - 08:55 trong Bất đẳng thức và cực trị

Cho a,b,c là các số thực phân biệt. Tìm giá trị nhỏ nhất của biểu thức.

$P=\sum a^2 + \sum \frac{1}{(a-b)^2}$

Spoiler

 




#619204 $\frac{1}{1+S_1}+\sum_{i=0}^...

Đã gửi bởi ZzNightWalkerZz on 08-03-2016 - 21:09 trong Bất đẳng thức - Cực trị

Cho dãy $S_n$ như sau

$S_1=a_1+a_2+..., S_2=a_1 a_2+.... ,...., S_n=a_1 a_2 ... a_n$ Với $a_1 ,a_2 ....\in [0;1]$

Chứng minh  bất đẳng thức sau :

$\frac{1}{1+S_1}+\sum_{i=0}^{n} \frac{1}{2i+2}.S_{2i+1}\leq 1+\sum_{i=1}^{n} \frac{1}{2i+1}.S_{2i}$




#586474 1+x+y không chia hết cho p với mọi p

Đã gửi bởi ZzNightWalkerZz on 31-08-2015 - 20:56 trong Số học

a) Nếu $p$ nguyên tố thì suy ra $x,y\vdots p=>1+x+y\not\vdots p$

b) Nếu $p$ là hợp số. Gọi $k$ là ước nguyên tố của $p$ 

Theo a) thì $x,y\vdots k=>x+y+1\not \vdots k=>x+y+1\not \vdots p$

Vậy....




#585732 CM: $\forall n\in mathbb{N}$ thì $B= 9n^3...

Đã gửi bởi ZzNightWalkerZz on 29-08-2015 - 15:23 trong Số học

Đặt $P(n)=n+1)(n+2)(n+3)......(n+n)$
Ta có $P(1)=2\vdots 2^1$ (đúng).Giả sử $P(n)\vdots 2^n$ với mọi $n$ là số tự nhiên
Lại có: $P(n+1)=P(n).2\vdots 2^n.2=2^{n+1}$
Vậy theo nguyên lí qui nạp ta có đpcm

Ở dòng thứ 3 phải là : $P(n+1)=P(n).2(2n+1)$ 




#584965 Tìm $max A=x^2+y^2$

Đã gửi bởi ZzNightWalkerZz on 25-08-2015 - 22:03 trong Bất đẳng thức và cực trị

Áp dụng AM-GM ta có $4=x^2+y^2-xy \geq x^2+y^2- \frac{x^2+y^2}{2}=\frac{x^2+y^2}{2}$

Suy ra $8 \geq x^2+y^2$

Dấu ''='' xảy ra khi $x=y=-2$ hoặc $x=y=2$

Spoiler

Chỉ đơn giản là : $-xy\leq \frac{x^2+y^2}{2}$




#584739 CMR $\exists k\in\mathbb{Z}:\; 2^nk+1...

Đã gửi bởi ZzNightWalkerZz on 24-08-2015 - 21:20 trong Số học

n = 0 chọn k = ... 

n = 1 chọn k = ...

n > 1 chọn k >1 khi đó : $2^nk+1>3$ 

Do đó ta chỉ cần chọn k để số này chia hết cho 3 là được. 

Rõ ràng là ta có thể chọn được ( bạn tự giải nốt nhá )

:icon10:

Đề bài nói là với 1 giá trị của $k$ thì thỏa mãn tất cả giá trị của $n$ chứ, như trên thì là đối với từng $n$




#584301 Chứng minh rằng $(x^{2}+y^{2})^{n}\ge...

Đã gửi bởi ZzNightWalkerZz on 23-08-2015 - 12:36 trong Bất đẳng thức - Cực trị

Áp dụng BĐT Cosi ta có : $x^{2a}y^{2b}+x^{2b}y^{2a}\geq (xy)^{a+b}$

$=> (a+b)^n=a^n+C_n^1a^{n-1}b+...+C_n^1ab^{n-1}+b^n\geq a^n+b^n+(C_n^1+C_n^2+...+C_n^{n-1}).(ab)^{\frac{n}{2}}$ (Chỗ này ko biết nên viết thế nào cho dễ hiểu, thông cảm :v)

$=a^n+b^n+(2^n-2)(ab)^{\frac{n}{2}}$

Thay $a=x^2, b=y^2=>(x^{2}+y^{2})^{n}\geq x^{2n}+y^{2n}+(2^n-2)(xy)^n=2^nx^ny^n+(x^n-y^n)^2$




#583661 $3\left [ x^{2} \right ]+5\left [ x \right...

Đã gửi bởi ZzNightWalkerZz on 21-08-2015 - 17:42 trong Số học

Thật sự là phần này đúng ạ. Khi nào nào vậy mới sai nhá: $x^{2}=([x])^{2}$.

 

$\left [ x^{2} \right ]=\left [ x \right ].\left [ x \right ]$ ?

P/s: Mình đang phân vân đoạn này

Chỗ này thực sự chưa đúng : với $x=3,9$ thì $[x^2]=15, [x]^2=9$

Bài trên mình có cách giải riêng (Chưa xem tài liệu)

Ta luôn có : $x^2\geq [x]^2$ mà $[x^2]$ là số nguyên dương lớn nhất không quá $x^2$ nên $[x^2]\geq [x]^2$

$=> 2\geq 3[x]^2+5[x]<=>0\geq (3[x]-1)([x]+2)=>\frac{1}{3}\geq [x]\geq -2$

Đến đây coi như xong




#582892 CMR $a^{p-1}$ là số đầu tiên

Đã gửi bởi ZzNightWalkerZz on 18-08-2015 - 18:27 trong Số học

Chứng minh rằng $a^{p-1}$ là số đầu tiên trong dãy $a^i$ có số dư là 1 khi chia cho $p$ $(i=1,2...)$

Ý kiến




#582728 Tìm các số tự nhiên $a,b,c,d$ thoả mãn $ 4^a.5^b - 3^c.11^d =...

Đã gửi bởi ZzNightWalkerZz on 17-08-2015 - 22:18 trong Số học

Tìm các số tự nhiên $a,b,c,d$ thoả mãn $ 4^a.5^b - 3^c.11^d = 1.$

Một nghiệm của phương trình là $a=c=1, b=d=0$

Các trường hợp $a=0$ với $b=0$ có lẽ không cần nói đến

Với $d=0$

Nếu $c=0$ thì vô lí $=>c\neq 0=> c$ lẻ (ai đọc tự hiểu :D) $=> 3^c+1\not \vdots 5$ 

Xét với $d>0$ thì chỉ cần bổ đề sau là được 

Bổ đề

Giải nốt sẽ hơi nhàm  :icon6:

Lời giải còn sơ sài, thiếu chỗ nào mọi người thông cảm  :mellow:




#582706 CMR: $\sqrt{x+1}+\sqrt{y+1}+\sqrt...

Đã gửi bởi ZzNightWalkerZz on 17-08-2015 - 21:26 trong Bất đẳng thức và cực trị

Cho $x,y,z\in \left [ -1;1 \right ]$;$xy+yz+zx=0$.CMR: $\sqrt{x+1}+\sqrt{y+1}+\sqrt{z+1}\leq 3$

Lấy $x=y=0, z=1$ hiển nhiên điều phải chứng minh không đúng




#582699 TÌm GTLN,GTNN của S=(2-x)(2-y)

Đã gửi bởi ZzNightWalkerZz on 17-08-2015 - 21:10 trong Bất đẳng thức và cực trị

Mình mới tìm ra min thôi:

Ta có:

$2S= 8+2xy-4(x+y)\Leftrightarrow 2S+1= (x+y)^{2}-4(x+y)+8\geq 4$

$\Leftrightarrow S\geq \frac{3}{2}.$ :closedeyes:

Xem lại dấu "="

Biến đổi như trên thì ta chỉ cần phải tìm min, max của : $(x+y)^2-4(x+y)+4=(2-x-y)^2$

Dễ dàng nhận thấy $2>x+y$ nên ta chỉ phải tìm min, max của $2-x-y>0$

Mà $(x+y)^2=1+2xy\leq 1+\frac{(x+y)^2}{2}<=>(x+y)^2\leq 2<=> -\sqrt{2}\leq x+y\leq \sqrt{2}$

Vậy $S_{min}$ khi $x=y=\frac{\sqrt{2}}{2}$, $S_{max}$ khi $x=y=-\frac{\sqrt{2}}{2}$




#577601 Tính $M=x-y$

Đã gửi bởi ZzNightWalkerZz on 01-08-2015 - 21:42 trong Đại số

Dễ mờ, làm gì kinh khủng thế.

Cộng, trừ lần lượt hai đẳng thức ta có :

$\frac{2}{y}=y^2+3x^2;\frac{10}{x}=3y^2+x^2<=>10=3y^2x+x^3;2=3x^2y+y^3$ 

Trừ hai vế : $8=x^3-3x^2y+3y^2-y^3=(x-y)^3<=>x-y=2$




#577562 $\frac{1}{x_{1}}+\frac{2}{x_{1}+x_{2}}+...+\frac{n}{...

Đã gửi bởi ZzNightWalkerZz on 01-08-2015 - 20:20 trong Bất đẳng thức và cực trị

Đề ban đầu sai thì phải, cho $x_1\geq x_2...\geq x_n$ thì bài toán đổi chiều




#576917 Chứng minh rằng $n$ có không ít hơn một ước số nguyên tố thì ...

Đã gửi bởi ZzNightWalkerZz on 30-07-2015 - 21:41 trong Số học

Đề bài chưa đúng cho lắm, phải là $n$ có trên 1 ước số nguyên tố mới đúng

$\boxed{\text{Chiều thuận}}$

Do $n$ có trên 1 ước số nguyên tố nên luôn phân tích $n$ thành tích $p.q$ với $(p,q)=1$

Do $(k,k+1)=1$ nên ta phải chứng minh tồn tại số nguyên $k$ thỏa mãn hai điều kiện

$$k=pk_1;k+1=qk_2$$

Giả sử trong dãy số : $q,2q,3q...(p-1)q$ có 2 số có cùng số dư khi chia cho $p$. Giả sử hai số đó là $m_1q, m_2q$ ($1\leq m_1<m_2\leq p-1$)

$=>(m_2-m_1)q\vdots p$ (Vô lí do $(p,q)=1, m_2-m_1<p$)

Vậy trong $p-1$ số trên không có số nào có cùng số dư khi chia cho $p$ nên tồn tại ít nhất một số chia cho $p$ dư 1, ta đã chứng minh được tồn tại số nguyên tố $k$ thỏa mãn.

$\boxed{\text{Chiều nghịch}}$

Giả sử $n$ chỉ có một ước số nguyên tố, khi đó $n$ có dạng $n=p^x$ với $p$ là một số nguyên tố

Do $(k,k+1)=1$ nên chỉ có một số chia hết cho $n$ nên $k\geq n$ hoặc $k+1\geq n$ (Vô lí)

Vậy $n$ có trên 1 ước số nguyên tố

 




#576360 $p.x^2+q.y^2=m.z^2$

Đã gửi bởi ZzNightWalkerZz on 28-07-2015 - 22:49 trong Số học

Giải phương trình $p.x^2+q.y^2=m.z^2$ với $x,y,z$ là các số tự nhiên, $p,q,m$ là các số nguyên tố cho trước.

.




#576351 $2x^{2x}-1=y^{z+1}$

Đã gửi bởi ZzNightWalkerZz on 28-07-2015 - 22:30 trong Số học

Thử với $x=1$ thì $y^{z+1}=3$ suy ra $y=3,z=0$

Mình nghĩ là không có nghiệm nguyên dương thì đúng hơn

Hình như anh nhầm đề

Là $2x^{2x}-1=y^{z+1}$ chứ không phải $(2x)^{2x}-1=y^{z+1}$




#574331 $2^m+1$ không chia hết cho $2^n-1$

Đã gửi bởi ZzNightWalkerZz on 20-07-2015 - 21:36 trong Số học

$\texttt{Solution}$

 

$\blacklozenge$ Nếu $m<n$ thì với $n>2$ ta có : $2^m+1<2^n-1$

Suy ra $2^m+1$ không chia hết cho $2^n-1$

 

$\blacklozenge$ Nếu $m=n$ thì $\frac{2^m+1}{2^n-1}=1+\frac{2}{2^n-1}$ 

Với $n>2$ thì  $\frac{2^m+1}{2^n-1}=1+\frac{2}{2^n-1}$  không phải là số nguyên. 

Suy ra $2^m+1$ không chia hết cho $2^n-1$

 

$\blacklozenge$ Nếu $m>n$. Ta đặt $m=kn+h$ ($k,h$ nguyên dương, $r$ tự nhiên và nhỏ hơn $n$.

Khi đó : $\frac{2^m+1}{2^n-1}=\frac{2^m-2^r}{2^n-1}+\frac{2^r+1}{2^n-1}=\frac{2^m-2^{m-kn}}{2^n-1}+\frac{2^r+1}{2^n-1}=\frac{2^{m-kn}(2^{kn}-1)}{2^n-1}+\frac{2^r+1}{2^n-1}$

Nhận xét : $\frac{2^{m-kn}(2^{kn}-1)}{2^n-1}$ nguyên. Vì $r<n$ nên theo chứng minh trên thì $\frac{2^r+1}{2^n-1}$ không nguyên

Suy ra $2^m+1$ không chia hết cho $2^n-1$

 

Vậy trong mọi trường hợp thì $2^m+1$ không chia hết cho $2^n-1$    $\square$

Cách giải rất hay nhưng $h$ biến mất đâu rồi anh?  :icon6:




#573157 $\sum_{i=1}^{n}\sqrt{x_{i}...

Đã gửi bởi ZzNightWalkerZz on 16-07-2015 - 20:22 trong Bất đẳng thức - Cực trị

Giả thiết tương đương với $\sum \frac{x_i-(n-1)}{1+x_i}=0$

Bất đẳng thức cần chứng minh tương đương với $\sum \frac{x_i-(n-1)}{\sqrt{x_i}}\geq0$

Vậy thì bây giờ ta chỉ việc chứng minh $(\sum \frac{x_i-(n-1)}{\sqrt{x_i}})(\sum \frac{\sqrt{x_i}}{x_i+1})\geq0=\sum \frac{x_i-(n-1)}{1+x_i}$

BĐT trên chính là BĐT Chebyshev. Để thỏa mãn ta chỉ còn phải chứng minh

Với $\frac{x_i-(n-1)}{\sqrt{x_i}}$ tăng dần thì $\frac{\sqrt{x_i}}{x_i+1}$ giảm dần khi cho $i$ di động từ $1$ đến $n$ (Đơn giản nên mình không chứng minh)




#572847 $n^2+2^n\vdots 2p$

Đã gửi bởi ZzNightWalkerZz on 15-07-2015 - 20:51 trong Số học

$\texttt{Bài toán}$ Cho $p$ là số nguyên tố có dạng $p=4k+1$. Hỏi có tồn tại hay không số tự nhiên $n$ sao cho $n^2+2^n\vdots 2p$

 

Gợi ý




#572208 $\sum_{k=1}^{n^2}\left \{ \...

Đã gửi bởi ZzNightWalkerZz on 13-07-2015 - 23:06 trong Số học

$\texttt{Solution}$

Spoiler


Ta sẽ chứng minh khẳng định bằng quy nạp theo $n$.
Với $n=1$ thì hiển nhiên khẳng đinh đúng. Giả sử khẳng định đúng với $n$ , ta chứng minh khẳng định đúng với $n+1$. Ta có nhận xét :
$n<\sqrt{n^2+1}<\sqrt{n^2+2}<...<\sqrt{n^2+2n}<n+1$
nên $\forall i=1,2,..,2n$ ta có : $\left \{\sqrt{n^2+i} \right \} =\sqrt{n^2+i}-n<\sqrt{n^2+i+\frac{i^2}{4n^2}}-n=\frac{i}{2n}$
Ta có :
$\sum_{k=1}^{(n+1)^2}\left \{ \sqrt{k} \right \}=\sum_{k=1}^{n^2}\left \{ k \right \}+\sum_{k=n^2+1}^{(n+1)^2}\left \{ k \right \}$
$< \frac{n^2-1}{2}+\frac{\sum_{i=1}^{2n}i}{2n}$
$= \frac{n^2-1}{2}+\frac{2n+1}{2}=\frac{(n+1)^2-1}{2}$
Suy ra khẳng định đúng với $n+1$
Vậy bài toán được chứng minh $\square$

Chuẩn cách em. Đang định gửi thì a trả lời : D



#571866 $\sum \sqrt{a_i} \notin Q$

Đã gửi bởi ZzNightWalkerZz on 12-07-2015 - 21:50 trong Số học

Cái tổng quát kia không đúng đâu nhé. Ví dụ $\sqrt{3}-\sqrt{3}=0$, nhưng nếu cho thêm điều kiện khác 0 thì đề bài đúng. Nếu mà dùng lí thuyết Galois thì người ta còn chứng minh được một điều mạnh hơn nhiều là $\sum \sqrt{a_i}$ cùng với $\mathbb{Q}$ sinh ra tất cả các số $\sqrt{a_i}$. Chứng minh bằng lí thuyết Galois thì đơn giản lắm, nhưng cần phát biểu lại cho bản chất hơn tí: Cho $\alpha_{i}$ là các nghiệm của đa thức bất khả quy $x^2-a_i=0$ với $a_i \in \mathbb{R}$(không cần thiết phải không chính phương mà chỉ cần $a_i$ không là bình phương của một số hữu tỉ) và $\alpha=\sum \alpha_{i}$ khác 0 thì $\alpha \notin \mathbb{Q}$. Ta có thể giả sử rằng không có tổng con nào của tổng trên bằng 0 (bằng cách bỏ hết nó đi trong trường hợp nó tồn tại), ta có $\prod (x^2-a_i)$ là tách được nên ta có nhóm Galois G của đa thức này. Mỗi phần tử nhóm G được xác định hoàn toàn bởi các $\alpha_i$ (do trường phân rã của đa thức trên là $\mathbb{Q}(\alpha_1,...,\alpha_r)$), vì mỗi phần tử này giao hoán các nghiệm của đa thức $x^2-a_i$, một phần tử không tầm thường sẽ giao hoán một số nào đó $\alpha_i$ với $-\alpha_i$. Những phần tử như vậy không thể giữ nguyên $\alpha$ được vì nếu không sẽ có một tổng con nào đó bằng 0. Như vậy phần tử duy nhất giữ nguyên $\alpha$ là trung hòa của G, do đó $\mathbb{Q}(\alpha)$ là tương ứng với 1 trong G, tức là $\mathbb{Q}(\alpha)=\mathbb{Q}(\alpha_i)$, nói riêng thì $\alpha$ không thuộc $\mathbb{Q}$ được vì các $\alpha_i$ không thuộc $\mathbb{Q}$. Viết thì dài thế nhưng mà ý tưởng cơ bản là xem khi giao hoán $\alpha_i$ với $-\alpha_i$ thì $\alpha$ có thay đổi không.    

Đúng là rất đơn giản nhưng tổng quát không sai nhé anh, anh xem lại là m>n nên giá trị trên luôn khác không anh ạ, mà nói chính phương ở đây chính là bình phương một số hữu tỉ (do em lười không muốn viết đầy đủ thôi ạ)  :D

Nếu như bài này chỉ có phép cộng thì cách đưa đến giới hạn khá hữu hiệu nhưng dài, có lẽ cái gì đã có thì thôi để đó vậy :wub:

P/s : Em phải đọc kĩ hơn về lí thuyết này :(




#571801 $\sum \sqrt{a_i} \notin Q$

Đã gửi bởi ZzNightWalkerZz on 12-07-2015 - 18:24 trong Số học

Spoiler
Lý thuyết Galois toàn làm về mấy cái này mà. Em post cách chứng minh lên được không. Anh hơi tò mò.

Chưa được anh  :icon6: vì anh đã làm được bài trên nên anh thử nghĩ một bài tổng quát hơn nhé (Em chưa làm được theo cách ban đầu  :wacko: )
Điều kiện thì gần như  trên các số $a_i,b_e$ không là số chính phương ($i=1,2,..,n;e=1,2,...,m;m>n$)
Cm : $\sum \sqrt{a_i} - \sum \sqrt{b_e}\notin Q$
Nếu anh làm được bằng lí thuyết $Galois$ thì up lời giải nhé

Spoiler




#571740 $\sum_{k=1}^{n^2}\left \{ \...

Đã gửi bởi ZzNightWalkerZz on 12-07-2015 - 14:25 trong Số học

Russia 1999 : Chứng minh rằng với mọi số tự nhiên dương $n$ : $\sum_{k=1}^{n^2}\left \{ \sqrt{k} \right \}\leq \frac{n^2-1}{2}$

Cho em hỏi cái, $VT$ có $n^2$ số hạng nên $VT\geq n^2>VP$ rồi còn gì nữa ?  :mellow: